www.vorhilfe.de
Vorhilfe

Kostenlose Kommunikationsplattform für gegenseitige Hilfestellungen.
Hallo Gast!einloggen | registrieren ]
Startseite · Forum · Wissen · Kurse · Mitglieder · Team · Impressum
Forenbaum
^ Forenbaum
Status Englisch
  Status Grammatik
  Status Lektüre
  Status Korrekturlesen
  Status Übersetzung
  Status Sonstiges (Englisch)

Gezeigt werden alle Foren bis zur Tiefe 2

Navigation
 Startseite...
 Neuerdings beta neu
 Forum...
 vorwissen...
 vorkurse...
 Werkzeuge...
 Nachhilfevermittlung beta...
 Online-Spiele beta
 Suchen
 Verein...
 Impressum
Das Projekt
Server und Internetanbindung werden durch Spenden finanziert.
Organisiert wird das Projekt von unserem Koordinatorenteam.
Hunderte Mitglieder helfen ehrenamtlich in unseren moderierten Foren.
Anbieter der Seite ist der gemeinnützige Verein "Vorhilfe.de e.V.".
Partnerseiten
Weitere Fächer:

Open Source FunktionenplotterFunkyPlot: Kostenloser und quelloffener Funktionenplotter für Linux und andere Betriebssysteme
Forum "Integration" - Integration mit Betrag
Integration mit Betrag < Integration < Funktionen < eindimensional < reell < Analysis < Hochschule < Mathe < Vorhilfe
Ansicht: [ geschachtelt ] | ^ Forum "Integration"  | ^^ Alle Foren  | ^ Forenbaum  | Materialien

Integration mit Betrag: Frage (beantwortet)
Status: (Frage) beantwortet Status 
Datum: 11:19 Fr 22.05.2009
Autor: Pille456

Aufgabe
Bestimmen Sie den Wert folgender Integrale:
(i) [mm] \integral_{-1}^{1}{\bruch{1}{\sqrt{|x|}} dx} [/mm]

Hallo,
Ich habe gerade Probleme mit dem Betrag in der Funktion. Mein Ansatz sieht wie folgt aus:
Für [mm] x\ge0: \integral_{0}^{1}{\bruch{1}{\sqrt{|x|}} dx} [/mm] = [mm] [2*\sqrt{x}]_0^1 [/mm] = 2-0 = 2
Für [mm] x\le0: \integral_{-1}^{0}{\bruch{1}{\sqrt{|x|}} dx} [/mm] = [mm] [2*\sqrt{-x}]_{-1}^0 [/mm] = 0-2 = -2
Nun müsste ich die beiden Flächeninhalte Betragsweise addieren, also 2+2 = 4 für den Gesamtflächeninhalt.
Das kommt von der Anschauung her ganz gut hin, gerade weil die Funktion Achsensymetrisch ist.
Nur bin ich mir gerade nicht sicher ob das so mathematisch korrekt ist, gerade weil an der Stelle 0 die Funktion ja nicht definiert ist.
Oder wie geht man solche Betragsfunktionen beim Integrieren an?

P.S: Sei f eine Funktion die auf dem Intervall [a,b] definiert ist. So ist doch f' (also die Ableitung) auf (a,b) definiert. Also man leitet ja immer auf offene Intervalle ab, weil man eine "Umgebung" braucht um abzuleiten.
Heißt das im Umkehrschluss, dass man auf geschlossene Intervalle aufleitet, also auch in einem bestimmten Punkt integrieren kann?!

        
Bezug
Integration mit Betrag: Antwort
Status: (Antwort) fertig Status 
Datum: 11:27 Fr 22.05.2009
Autor: fred97


> Bestimmen Sie den Wert folgender Integrale:
>  (i) [mm]\integral_{-1}^{1}{\bruch{1}{\sqrt{|x|}} dx}[/mm]
>  Hallo,
>  Ich habe gerade Probleme mit dem Betrag in der Funktion.
> Mein Ansatz sieht wie folgt aus:
>  Für [mm]x\ge0: \integral_{0}^{1}{\bruch{1}{\sqrt{|x|}} dx}[/mm] =
> [mm][2*\sqrt{x}]_0^1[/mm] = 2-0 = 2
>  Für [mm]x\le0: \integral_{-1}^{0}{\bruch{1}{\sqrt{|x|}} dx}[/mm] =
> [mm][2*\sqrt{-x}]_{-1}^0[/mm] = 0-2 = -2



Das ist nicht korrekt !

[mm]x\le0: \integral_{-1}^{0}{\bruch{1}{\sqrt{|x|}} dx}[/mm] =  [mm][-2*\sqrt{-x}]_{-1}^0[/mm] = 0+2 = 2

Kettenregel !!

>  Nun müsste ich die beiden Flächeninhalte Betragsweise
> addieren, also 2+2 = 4 für den Gesamtflächeninhalt.
> Das kommt von der Anschauung her ganz gut hin, gerade weil
> die Funktion Achsensymetrisch ist.
>  Nur bin ich mir gerade nicht sicher ob das so mathematisch
> korrekt ist, gerade weil an der Stelle 0 die Funktion ja
> nicht definiert ist.



Eigentlich müßtest Du so vorgrhen:

Für [mm]x\ge0:\limes_{t\rightarrow 0+} \integral_{t}^{1}{\bruch{1}{\sqrt{|x|}} dx}[/mm]

Entprechend beim 2. Integral



>  Oder wie geht man solche Betragsfunktionen beim
> Integrieren an?
>  
> P.S: Sei f eine Funktion die auf dem Intervall [a,b]
> definiert ist. So ist doch f' (also die Ableitung) auf
> (a,b) definiert.

Quatsch ! Es gibt doch rechts- bzw linksseitige Ableitungen


> Also man leitet ja immer auf offene
> Intervalle ab, weil man eine "Umgebung" braucht um
> abzuleiten.
>  Heißt das im Umkehrschluss, dass man auf geschlossene
> Intervalle aufleitet,

Ah, ..... lass dieses Unwort "aufleiten"   !!!!!

> also auch in einem bestimmten Punkt
> integrieren kann?!

Unsinn !!


FRED

Bezug
                
Bezug
Integration mit Betrag: Mitteilung
Status: (Mitteilung) Reaktion unnötig Status 
Datum: 11:39 Fr 22.05.2009
Autor: M.Rex

Hallo fred

>  >  Heißt das im Umkehrschluss, dass man auf geschlossene
> > Intervalle aufleitet,
>
> Ah, ..... lass dieses Unwort "aufleiten"   !!!!!

Leider ist das inzwischen absolut üblich, dieses Wort zu benutzen, selbst im Klett-Duden fürs Abi 2009 taucht das Wort auf. Da hatten wir auf dem Teamtreffen auch eine Diskussion drüber. Es scheint so, als müssten wir uns daran gewöhnen.

>  

Marius


Bezug
                        
Bezug
Integration mit Betrag: Mitteilung
Status: (Mitteilung) Reaktion unnötig Status 
Datum: 11:50 Fr 22.05.2009
Autor: fred97


> Hallo fred
>  
> >  >  Heißt das im Umkehrschluss, dass man auf geschlossene

> > > Intervalle aufleitet,
> >
> > Ah, ..... lass dieses Unwort "aufleiten"   !!!!!
>  
> Leider ist das inzwischen absolut üblich, dieses Wort zu
> benutzen, selbst im Klett-Duden fürs Abi 2009 taucht das
> Wort auf.

Na und ? An Universitäten benutzt keiner der Mathematikdozenten dieses Wort. Dennen stehen die Haare zu Berge, wenn sie so was lesen oder hören




> Da hatten wir auf dem Teamtreffen auch eine
> Diskussion drüber. Es scheint so, als müssten wir uns daran
> gewöhnen.


Niemals

Gruß FRED



>  
> >  

>
> Marius
>  


Bezug
                                
Bezug
Integration mit Betrag: Mitteilung
Status: (Mitteilung) Reaktion unnötig Status 
Datum: 12:44 Fr 22.05.2009
Autor: Marcel

Hallo,

> > Hallo fred
>  >  
> > >  >  Heißt das im Umkehrschluss, dass man auf geschlossene

> > > > Intervalle aufleitet,
> > >
> > > Ah, ..... lass dieses Unwort "aufleiten"   !!!!!
>  >  
> > Leider ist das inzwischen absolut üblich, dieses Wort zu
> > benutzen, selbst im Klett-Duden fürs Abi 2009 taucht das
> > Wort auf.

Autsch!!

> Na und ? An Universitäten benutzt keiner der
> Mathematikdozenten dieses Wort. Dennen stehen die Haare zu
> Berge, wenn sie so was lesen oder hören

In der Tat, keiner meiner Dozenten hat dieses Wort auch nur einmal benutzt... Ach doch, einmal, um zu sagen, dass er es nie wieder hören will ;-)

> > Da hatten wir auf dem Teamtreffen auch eine
> > Diskussion drüber. Es scheint so, als müssten wir uns daran
> > gewöhnen.
>  
>
> Niemals

Das sehe ich auch so. Ich finde eine Diskussion darüber auch für Schüler nicht so sinnvoll, denn in der Schule bekommt man von manchen Lehrern leider auch vermittelt, dass das Wort eine Rechtfertigung hätte. In Wahrheit ist es eher so, dass dieses Wort in vielen Fällen eher zu Verwirrung als Klahrheit sorgt; nicht umsonst steht dazu auch ein Kommentar bei []Wiki:
"Das oft von Schülern gebrauchte Wort Aufleitung ist kein mathematischer Fachterminus und ist auch etymologisch in Bezug auf den Begriff Ableitung nicht zu rechtfertigen. Es ist mathematisch nicht korrekt, das Finden einer Stammfunktion als bloße Umkehrung des Ableitens zu betrachten. Der genaue Zusammenhang kann nicht kürzer als im Hauptsatz formuliert werden."

Gruß,
Marcel

Bezug
                
Bezug
Integration mit Betrag: Frage (beantwortet)
Status: (Frage) beantwortet Status 
Datum: 11:42 Fr 22.05.2009
Autor: Pille456

Hm okay das mit dem Integral macht soweit Sinn für mich - danke!!
Nun zu der anderen Sache:
Mir wurde es so erklärt(Wobei der Prof. dazu meinte, es sei eine "akademische Feinheit" wie genau man das definiert und da er der Prof. ist und ich der Student habe ich das erstmal so hingenommen, da es sich auch recht schlüssig für mich anhörte.Also nicht steinigen wenn ich hier nun Schrott erzähle...)
Wenn f auf [a,b] definiert ist und f differenzierbar ist, dann gilt schonmal, dass f' auf (a,b) differenzierbar ist.(sofern ich kritische Punkte wie z.B. die 0 bei f(x) = |x| überprüft habe)
In den Punkten a und b muss ich die Funktion dann gesondert untersuchen und schauen ob ich dort eine Ableitung bilden kann.
Das heißt für mich nun, wenn ich f' integriere, also f bilde, dass ich dann wieder auf ein geschlossenes Intervall [a,b] komme

Bezug
                        
Bezug
Integration mit Betrag: Antwort
Status: (Antwort) fertig Status 
Datum: 13:14 Fr 22.05.2009
Autor: Marcel

Hallo,

> Hm okay das mit dem Integral macht soweit Sinn für mich -
> danke!!
>  Nun zu der anderen Sache:
>  Mir wurde es so erklärt(Wobei der Prof. dazu meinte, es
> sei eine "akademische Feinheit" wie genau man das definiert
> und da er der Prof. ist und ich der Student habe ich das
> erstmal so hingenommen, da es sich auch recht schlüssig für
> mich anhörte.Also nicht steinigen wenn ich hier nun Schrott
> erzähle...)
>  Wenn f auf [a,b] definiert ist und f differenzierbar ist,
> dann gilt schonmal, dass f' auf (a,b) differenzierbar
> ist.(sofern ich kritische Punkte wie z.B. die 0 bei f(x) =
> |x| überprüft habe)

ach Du Schande. Es gibt differenzierbare Funktionen [mm] $f\,$, [/mm] deren Ableitung [mm] $f\,\!'$ [/mm] nicht stetig ist; die Ableitungen sind dann insbesondere nicht differenzierbar (Diff'barkeit impliziert Stetigkeit!). Nicht umsonst gibt es in der Analysis die Klasse der (natürlich bzgl. eines gemeinsamen, entsprechenden Definitionsbereichs (und Wertebereichs)) stetig differenzierbaren Abbildungen, welche die Menge aller differenzierbaren Abbildungen mit stetigen Ableitungen enthält (beachte bitte, dass die Ableitung stetig sein soll; denn bei einer differenzierbaren Funktion ist die Funktion selbst insbesondere stetig!). Warum bräuchte man sowas, wenn differenzierbare Abbildungen auch eine differenzierbare Ableitung hätten? Da hat Dein Prof. aber sehr groben Unfug erzählt, sorry, wenn ich so hart ins Gericht mit ihm gehe, aber das, was da oben steht, ist echt totaler Unsinn.

>  In den Punkten a und b muss ich die Funktion dann
> gesondert untersuchen und schauen ob ich dort eine
> Ableitung bilden kann.
> Das heißt für mich nun, wenn ich f' integriere, also f
> bilde, dass ich dann wieder auf ein geschlossenes Intervall
> [a,b] komme

Keine Ahnung, was nun die Strategie hier sein soll, benutze lieber die von Fred vorgeschlagene Vorgehensweise:
Berechne
[mm] $$\lim_{\substack{a \to 0\\a > 0}}\int_{a}^1 \frac{1}{\sqrt{|x|}}\;dx=\lim_{\substack{a \to 0\\a > 0}}\int_{a}^1 \frac{1}{\sqrt{x}}\;dx\,,$$ [/mm]
und weil die Funktion $x [mm] \mapsto \frac{1}{\sqrt{|x|}}$ [/mm] ($x [mm] \not=0$) [/mm] symmetrisch (bzgl. der [mm] $y\,$-Achse) [/mm] ist, kannst Du danach
[mm] $$\lim_{\substack{b \to 0\\b < 0}}\int_{-1}^b \frac{1}{\sqrt{|x|}}\;dx\underset{\substack{\text{Subst. }x=-y\\dx=-dy\\x=1 \gdw y=-1\\x=b \gdw y=-b}}{=}\lim_{\substack{b \to 0\\b < 0}}\;-\;\int_{-(-1)}^{-b} \frac{1}{\sqrt{|-y|}}\;dy=\lim_{\substack{b \to 0\\b < 0}}\int_{-b}^{1} \frac{1}{\sqrt{|x|}}\;dx=\lim_{\substack{a \to 0\\a > 0}}\int_{a}^{1} \frac{1}{\sqrt{|x|}}\;dx$$ [/mm]
benützen.

Gruß,
Marcel

Bezug
                        
Bezug
Integration mit Betrag: Antwort
Status: (Antwort) fertig Status 
Datum: 13:27 Fr 22.05.2009
Autor: M.Rex

Hallo

Für den "Gerichteten Grenzwertübergang" [mm] a\to0 [/mm] mit a>0 (Also von oben) kannst du dann auch die Folge [mm] \alpha:=0+\bruch{1}{n}=\bruch{1}{n} [/mm] nehmen, und dann [mm] n\to\infty [/mm] laufen lassen, analog dann beim zweiten integral [mm] \beta:=0\red{-}\bruch{1}{n}=-\bruch{1}{n} [/mm]

Manchmal (ich finde sogar meistens) ist es einfacher, einen Laufindex gegen [mm] \infty [/mm] laufen zu lassen.

Marius

Bezug
                                
Bezug
Integration mit Betrag: Mitteilung
Status: (Mitteilung) Reaktion unnötig Status 
Datum: 13:37 Fr 22.05.2009
Autor: Marcel

Hallo,

> Hallo
>  
> Für den "Gerichteten Grenzwertübergang" [mm]a\to0[/mm] mit a>0 (Also
> von oben) kannst du dann auch die Folge
> [mm]\alpha:=0+\bruch{1}{n}=\bruch{1}{n}[/mm] nehmen, und dann
> [mm]n\to\infty[/mm] laufen lassen,

das ist auch so eine Sache:
Wenn man etwas weiß, dann geht das (hier klappt das!). Wenn man etwas nicht weiß, dann berechnet man evtl. einen 'Wert' und behauptet, dass das Integral diesen habe, in Wahrheit aber das Integral gar nicht existiert. Wenn man hier mit Folgen arbeiten wollte, müßte man eigtl. erstmal sagen:
Man nehme eine beliebige Folge [mm] $(\alpha_n)$ [/mm] in [mm] $\IR_{>0}$ [/mm] mit [mm] $\alpha_n \to \infty$ [/mm] (d.h. [mm] $(a_n)$ [/mm] soll nur durch diese Eigenschaften ausgezeichnet sein und es soll keine spezielle (bzw. konkrete) Folge hergenommen werden) und untersucht dann, ob [mm] $\lim_{n \to \infty} \int_{1/\alpha_n}^1 \frac{1}{\sqrt{|x|}}\;dx$ [/mm] existiert. (Um zu prüfen, ob [mm] $\lim_{\substack{a \to 0\\a > 0}}\int_{a}^1 \frac{1}{\sqrt{|x|}}\;dx$ [/mm] existiert und zur Angabe dieses Grenzwertes im Falle der Existenz. Denn beachte:
[mm] $\int_{0+}^1 \frac{1}{\sqrt{|x|}}\;dx$ [/mm] existiert genau dann, wenn für alle Folgen [mm] $(\alpha_n)_{n \in \IN}$, [/mm] die die Eigenschaften haben: [mm] $\alpha_n [/mm] > 0$ für jedes $n [mm] \in \IN$ [/mm] und [mm] $\alpha_n \to \infty$, [/mm] gilt, dass [mm] $\int_{1/\alpha_n}^1 \frac{1}{\sqrt{|x|}}\;dx$ [/mm] existiert.)

P.S.:
Z.B. würde Deine Vorgehensweise versagen, wenn Du
[mm] $$\int_0^\infty \sin(2\pi x)\;dx$$ [/mm]
berechnen würdest (ich übertrage es mal, was Du dann vermutlich machen wolltest:)
Hier wäre bspw. [mm] $\int_{0}^n \sin(2\pi x)\;dx=0$ [/mm] für alle $n [mm] \in \IN$, [/mm] also [mm] $\lim_{n \to \infty} \int_0^n \sin(2 \pi x)\;dx=\lim_{n \to \infty}0=0\,,$ [/mm]
[mm] $$\int_0^\infty \sin(2\pi x)\;dx$$ [/mm]
existiert nicht.

Gruß,
Marcel

Bezug
                                        
Bezug
Integration mit Betrag: Frage (beantwortet)
Status: (Frage) beantwortet Status 
Datum: 14:27 Fr 22.05.2009
Autor: Pille456

Hi Marcel
Ich muss/möchte mich etwas korrigieren bzw. schärfer ausdrücken. Folgendes habe ich mitgeschrieben:
"Um die Ableitung einer Funktion in einem Punkt zu bilden muss die Umgebung um diesen Punkt bekannt sein, das heißt der Links-und Rechtsseitige Grenzwert des Differenzenquotienten muss übereinstimmen.
Wenn f auf [a,b] definiert ist, dann ist f' auf (a,b) definiert, da man am Rand der Funktion (in den Punkten a oder b) die Umgebung nicht kennt."
Daher (so bin ich da jedenfalls von ausgegangen) muss man Funktionen an den Rändern des Intervalls z.B. gesondert auf Extrema untersuchen.
Und daher dann meine Schlussfolgerung: Wenn ich ableite so komme ich auf offene Intervalle. Wenn ich dann integriere muss ich doch wieder auf geschlossene Intervalle kommen...

Hm das mit den Folgen habe ich mal gekonnt überlesen ;) ich bin nun so vorgegangen, wie fred vorgeschlagen hat.

Bezug
                                                
Bezug
Integration mit Betrag: Antwort
Status: (Antwort) fertig Status 
Datum: 14:58 Fr 22.05.2009
Autor: Marcel

Hallo,

> Hi Marcel
>  Ich muss/möchte mich etwas korrigieren bzw. schärfer
> ausdrücken. Folgendes habe ich mitgeschrieben:
>  "Um die Ableitung einer Funktion in einem Punkt zu bilden
> muss die Umgebung um diesen Punkt bekannt sein, das heißt
> der Links-und Rechtsseitige Grenzwert des
> Differenzenquotienten muss übereinstimmen.
> Wenn f auf [a,b] definiert ist, dann ist f' auf (a,b)
> definiert, da man am Rand der Funktion (in den Punkten a
> oder b) die Umgebung nicht kennt."

er meint wohl, dass man i.a. nicht alle Funktionswerte von [mm] $f\,$ [/mm] auf einer Umgebung von [mm] $a\,$ [/mm] bzw. [mm] $b\,$ [/mm] kennt. Natürlich kann man sagen, dass, wenn [mm] $f\,$ [/mm] auf [mm] $[a,b]\,$ [/mm] definiert ist, mann dann die Ableitung [mm] $f\!\,'$ [/mm] bzgl. [mm] $(a,b)\,$ [/mm] betrachtet, aber wie Fred schon sagte: Man kann auch von [mm] $f\!\,'$ [/mm] auf [mm] $[a,b]\,$ [/mm] sprechen, wenn [mm] $f\!\,'(a+)$ [/mm] und [mm] $f\!\,'(b-)$ [/mm] existieren.

(Bei Dir oben macht's aber wenig Sinn, von $f'(0+)$ zu sprechen... Warum?)

>  Daher (so bin ich da jedenfalls von ausgegangen) muss man
> Funktionen an den Rändern des Intervalls z.B. gesondert auf
> Extrema untersuchen.

Verstehe ich nicht. Was hat das überhaupt mit Extrema zu tun?

>  Und daher dann meine Schlussfolgerung: Wenn ich ableite so
> komme ich auf offene Intervalle. Wenn ich dann integriere
> muss ich doch wieder auf geschlossene Intervalle kommen...

Verstehe ich auch nicht. Du wirst Dich später auch nochmal über Deine Gedankengänge hier wundern, wenn Du mit Lebesgue-Integralen rechnest. Dann hat man, unter gewissen Voraussetzungen, Gleichheiten wie
[mm] $$\int_{[a,b]}f=\int_{(a,b]}f=\int_{[a,b)}f=\int_{(a,b)}f=\int_{[a,b] \setminus N}f\,,$$ [/mm]
wobei [mm] $N\,$ [/mm] eine Lebesguesche Nullmenge ist (z.B. $N=[a,b] [mm] \cap \IQ$). [/mm]
  

> Hm das mit den Folgen habe ich mal gekonnt überlesen ;)

Wieso? Noch nicht bekannt?

> ich
> bin nun so vorgegangen, wie fred vorgeschlagen hat.

Gut ;-)

Gruß,
Marcel

Bezug
                                                        
Bezug
Integration mit Betrag: Frage (überfällig)
Status: (Frage) überfällig Status 
Datum: 15:09 Fr 22.05.2009
Autor: Pille456


> >  Daher (so bin ich da jedenfalls von ausgegangen) muss man

> > Funktionen an den Rändern des Intervalls z.B. gesondert auf
> > Extrema untersuchen.
>  
> Verstehe ich nicht. Was hat das überhaupt mit Extrema zu
> tun?

Das Thema mit dem dem Ableiten auf offene/geschlossene Intervalle kam in der Übung dran und dort wurde halt auch gesondert gesagt, dass man die Ränder der Funktion gesondert überprüfen muss, wenn man herausbekommt dass dort ein Extremum liegen könnte.

> Verstehe ich auch nicht. Du wirst Dich später auch nochmal
> über Deine Gedankengänge hier wundern, wenn Du mit
> Lebesgue-Integralen rechnest. Dann hat man, unter gewissen
> Voraussetzungen, Gleichheiten wie
>  
> [mm]\int_{[a,b]}f=\int_{(a,b]}f=\int_{(a,b]}f=\int_{(a,b)}f=\int_{[a,b] \setminus N}f\,,[/mm]
>  
> wobei [mm]N\,[/mm] eine Lebesguesche Nullmenge ist (z.B. [mm]N=[a,b] \cap \IQ[/mm]).

Hm sagt mir nichts, aber hört sich interessant an. Der Gedankengang war einfach der:
Wenn f auf [a,b] definiert ist und f' auf (a,b), dann müsste doch die Stammfunktion von f' (also f) auf [a,b] definiert sein, obwohl f' auf (a,b) definiert ist

> > Hm das mit den Folgen habe ich mal gekonnt überlesen ;)
>
> Wieso? Noch nicht bekannt?

Doch schon, aber empfand ich das eher als eine mathematische Feinheit die nicht unbedingt Sinn der Aufgabe ist. Also da wir gerade das Thema Integrale in der Vorlesung behandeln wollte ich auch die Aufgaben entsprechend (hinsichtlich der Klausur) lösen. Es ist cool, wenn man sowas weiss/erkennt, aber erstmal ja nicht unbedingt Pflicht zu wissen (jedenfalls von dem was die Vorlesung abverlangt ;)). Und da du ja auch ein Beispiel für Einschränkungen gebracht hast, habe ich die Aufgaben erstmal anders gelöst.

P.S: Zu der Funktion     $ [mm] \int_0^\infty \sin(2\pi x)\;dx [/mm] $. sin ist ja periodisch und hat somit unendlich viele Nullstellen. Daher reicht es doch eigentlich den Flächeninhalt von z.B. 0 bis zur 1. Nullstelle zu bestimmen (welche ist das genau? ich "sehe" es gerade nicht) und diesen dann einfach zu immer drauf zu addieren (natürlich Betragsweise für den Teil des Graphen der unter der x-achse liegt). Dann wäre (jedenfalls von der Anschauung her) der Flächeninhalt doch unendlich groß...

Bezug
                                                                
Bezug
Integration mit Betrag: Antwort
Status: (Antwort) fertig Status 
Datum: 16:38 Fr 22.05.2009
Autor: Marcel

Hallo,

nur kurz hierzu:

> P.S: Zu der Funktion     [mm]\int_0^\infty \sin(2\pi x)\;dx [/mm].

Das ist keine Funktion, sondern da steht ein Integral!

> sin ist ja periodisch und hat somit unendlich viele
> Nullstellen. Daher reicht es doch eigentlich den
> Flächeninhalt von z.B. 0 bis zur 1. Nullstelle zu bestimmen
> (welche ist das genau? ich "sehe" es gerade nicht) und
> diesen dann einfach zu immer drauf zu addieren (natürlich
> Betragsweise für den Teil des Graphen der unter der x-achse
> liegt). Dann wäre (jedenfalls von der Anschauung her) der
> Flächeninhalt doch unendlich groß...

Ne, das obige Integral existiert nicht. Die Menge der Nullstellen der Funktion $x [mm] \mapsto \sin(2\pi [/mm] x)$ - eingeschränkt auf [mm] $[0,\infty)$ [/mm] - ist gerade [mm] $\frac{1}{2}*\IN_0:=\Big\{\frac{1}{2}*n:\;n \in \IN_0\Big\}\,.$ [/mm]
Jetzt sieht man z.B.
[mm] $$\int_{0}^{\frac{1}{2}+n} \sin(2\pi x)\;dx=\int_0^{1/2} \sin(2\pi x)\;dx [/mm] = [mm] \text{const} [/mm] > [mm] 0\;\;\;\text{ für jedes }n \in \IN_0\,,$$ [/mm]
(wobei man hier die Konstante [mm] $\text{const}$ [/mm] auch konkreter angeben kann!)
aber
[mm] $$\int_{0}^{n} \sin(2\pi x)\;dx=\int_0^{1} \sin(2\pi x)\;dx=0\;\;\;\text{ für jedes }n \in \IN_0\,.$$ [/mm]

Damit existiert [mm]\int_0^\infty \sin(2\pi x)\;dx [/mm] nicht.

P.S.:
Du wirfst hier auch 'Fläche zwischen dem Graphen und der [mm] $x\,$-Achse' [/mm] und das Integral durcheinander. Z.B. ist [mm] $\int_\pi^{2\pi} \sin(x)\;dx$ [/mm] negativ, nur vom Betrage her stimmt das anschaulich mit dem Flächeninhalt zwischen dem Graphen von $x [mm] \mapsto \sin(x)$ [/mm] - eingeschränkt auf [mm] $[\pi,\;2\pi]$ [/mm] - überein (diese Fläche wäre also [mm] $\Big|\int_\pi^{2\pi} \sin(x)\;dx\Big|$). [/mm] Integrale können durchaus auch negative Werte annehmen!

Gruß,
Marcel

Bezug
                                                                
Bezug
Integration mit Betrag: Fälligkeit abgelaufen
Status: (Mitteilung) Reaktion unnötig Status 
Datum: 15:20 So 24.05.2009
Autor: matux

$MATUXTEXT(ueberfaellige_frage)
Bezug
Ansicht: [ geschachtelt ] | ^ Forum "Integration"  | ^^ Alle Foren  | ^ Forenbaum  | Materialien


^ Seitenanfang ^
www.englischraum.de
[ Startseite | Forum | Wissen | Kurse | Mitglieder | Team | Impressum ]